How to Find Tangent Lines to Y=x^2 Passing Through (2,3) [SOLVED] Tangent Line

Click For Summary
To find the tangent lines to the curve y=x^2 that pass through the point (2,3), the derivative y' = 2x is used to determine the slope of the tangent line at any point on the curve. The challenge is identifying the correct point on the curve where the tangent line intersects, as the slope must be calculated at that specific point. By setting up the equation of the tangent line using the slope and the point (2,3), the solution can be derived. The discussion highlights the importance of correctly identifying the tangent point to find the appropriate slope. Ultimately, the problem is solved by applying these principles to derive the tangent lines.
physstudent1
Messages
267
Reaction score
1
[SOLVED] Tangent Line

Homework Statement


Find all tangent lines that are tangent to Y=x^2 and go through the point (2,3)


Homework Equations





The Attempt at a Solution



At first I thought I knew how to do this problem, I found the derivative of the equation got:

y' = 2x ; then I plugged in x=2 to find the slope of the tangent however then i realized this is not the correct point to plug in because this is not the point that the line is actually going to be hitting the original graph at; I don't know how you would find this said point to plug in for x to get the slope can anyone steer me in the right direction
 
Physics news on Phys.org
You are looking for a line. You have the slope, and a point that the line goes through. That's sufficient information to derive the line equation.
 
I don't have the slope though because I don't know what point the line is tangent to on the curve so how can i plug a value into the derivative to get slope
 
y' = 2x is the family of all lines that are tangent to x^2. You are supposed to find the one that goes through (2,3).
 
oh i see thank you for your help :)
 
Question: A clock's minute hand has length 4 and its hour hand has length 3. What is the distance between the tips at the moment when it is increasing most rapidly?(Putnam Exam Question) Answer: Making assumption that both the hands moves at constant angular velocities, the answer is ## \sqrt{7} .## But don't you think this assumption is somewhat doubtful and wrong?

Similar threads

  • · Replies 1 ·
Replies
1
Views
999
  • · Replies 6 ·
Replies
6
Views
1K
  • · Replies 11 ·
Replies
11
Views
5K
  • · Replies 6 ·
Replies
6
Views
2K
  • · Replies 1 ·
Replies
1
Views
1K
  • · Replies 5 ·
Replies
5
Views
2K
  • · Replies 6 ·
Replies
6
Views
2K
  • · Replies 9 ·
Replies
9
Views
2K
Replies
13
Views
4K
  • · Replies 2 ·
Replies
2
Views
2K